Teorema do valor final

Em análise matemática, o teorema do valor final (FVT) é um dos vários teoremas semelhantes usados para relacionar expressões no domínio da frequência ao comportamento no domínio do tempo conforme o tempo se aproxima do infinito.[1][2][3][4] Matematicamente, se f ( t ) {\displaystyle f(t)} em tempo contínuo tem transformada de Laplace (unilateral) F ( s ) {\displaystyle F(s)} então, um teorema do valor final estabelece as condições sob as quais

lim t f ( t ) = lim s 0 s F ( s ) {\displaystyle \lim _{t\to \infty }f(t)=\lim _{s\,\to \,0}{sF(s)}}

Da mesma forma, se f [ k ] {\displaystyle f[k]} em tempo discreto tem transformada Z (unilateral) F ( z ) {\displaystyle F(z)} então, um teorema do valor final estabelece as condições sob as quais

lim k f [ k ] = lim z 1 ( z 1 ) F ( z ) {\displaystyle \lim _{k\to \infty }f[k]=\lim _{z\to 1}{(z-1)F(z)}}

Um teorema do valor final Abeliano faz suposições sobre o comportamento no domínio do tempo de f ( t ) {\displaystyle f(t)} (ou f [ k ] {\displaystyle f[k]} ) calcular lim s 0 s F ( s ) {\displaystyle \lim _{s\,\to \,0}{sF(s)}} . Por outro lado, um teorema do valor final de Tauber faz suposições sobre o comportamento no domínio da frequência de F ( s ) {\displaystyle F(s)} calcular lim t f ( t ) {\displaystyle \lim _{t\to \infty }f(t)} (ou lim k f [ k ] {\displaystyle \lim _{k\to \infty }f[k]} ) (ver teoremas Abeliano e Tauberiano para transformadas integrais ).

Teoremas de valor final para a transformada de Laplace

Deduzindo lim t f ( t ) {\displaystyle \lim _{t\to \infty }f(t)}

Nas seguintes declarações, a notação ' s 0 {\displaystyle s\to 0} ' significa que s {\displaystyle s} se aproxima de 0, enquanto ' s 0 {\displaystyle s\downarrow 0} ' significa que s {\displaystyle s} aproxima-se de 0 por meio dos números positivos.

Teorema do valor final estendido

Supondo que cada polo de F ( s ) {\displaystyle F(s)} está no meio plano esquerdo aberto ou na origem, e que F ( s ) {\displaystyle F(s)} tem no máximo um único polo na origem. Então s F ( s ) L R {\displaystyle sF(s)\to L\in \mathbb {R} } Como s 0 {\displaystyle s\to 0} e lim t f ( t ) = L {\displaystyle \lim _{t\to \infty }f(t)=L} .[5]

Teorema do valor final usando a transformada de Laplace da derivada

Supondo que f ( t ) {\displaystyle f(t)} e f ( t ) {\displaystyle f'(t)} ambos têm transformações de Laplace que existem para todos s > 0 {\displaystyle s>0} . E se lim t f ( t ) {\displaystyle \lim _{t\to \infty }f(t)} existe e lim s 0 s F ( s ) {\displaystyle \lim _{s\,\to \,0}{sF(s)}} existe então lim t f ( t ) = lim s 0 s F ( s ) {\displaystyle \lim _{t\to \infty }f(t)=\lim _{s\,\to \,0}{sF(s)}} . [3] [4] [6]

Observação

Ambos os limites devem existir para que o teorema seja válido. Por exemplo, se f ( t ) = sin ( t ) {\displaystyle f(t)=\sin(t)} então lim t f ( t ) {\displaystyle \lim _{t\to \infty }f(t)} não existe, mas lim s 0 s F ( s ) = lim s 0 s s 2 + 1 = 0 {\displaystyle \lim _{s\,\to \,0}{sF(s)}=\lim _{s\,\to \,0}{\frac {s}{s^{2}+1}}=0} . [3]:Example 2.37 [4]:20

Teorema do valor final do converso tauberiano aprimorado

Supondo que f : ( 0 , ) C {\displaystyle f:(0,\infty )\to \mathbb {C} } é limitado e diferenciável, e que t f ( t ) {\displaystyle tf'(t)} também é limitado por ( 0 , ) {\displaystyle (0,\infty )} . E se s F ( s ) L C {\displaystyle sF(s)\to L\in \mathbb {C} } Como s 0 {\displaystyle s\to 0} então lim t f ( t ) = L {\displaystyle \lim _{t\to \infty }f(t)=L} .[7]

Teorema do valor final de Abel

Supondo que cada polo de F ( s ) {\displaystyle F(s)} está no meio plano esquerdo aberto ou na origem. Em seguida, ocorre um dos seguintes:

  1. s F ( s ) L R {\displaystyle sF(s)\to L\in \mathbb {R} } Como s 0 {\displaystyle s\downarrow 0} e lim t f ( t ) = L {\displaystyle \lim _{t\to \infty }f(t)=L} .
  2. s F ( s ) + R {\displaystyle sF(s)\to +\infty \in \mathbb {R} } Como s 0 {\displaystyle s\downarrow 0} e f ( t ) + {\displaystyle f(t)\to +\infty } Como t {\displaystyle t\to \infty } .
  3. s F ( s ) R {\displaystyle sF(s)\to -\infty \in \mathbb {R} } Como s 0 {\displaystyle s\downarrow 0} e f ( t ) {\displaystyle f(t)\to -\infty } Como t {\displaystyle t\to \infty } .

Em particular, se s = 0 {\displaystyle s=0} é um polo múltiplo de F ( s ) {\displaystyle F(s)} então o caso 2 ou 3 se aplica ( f ( t ) + {\displaystyle f(t)\to +\infty } ou f ( t ) {\displaystyle f(t)\to -\infty } ) [5]

Supondo que a transformada de Laplace de f ( t ) {\displaystyle f(t)} existe. Tomando λ > 1 {\displaystyle \lambda >-1} . Se lim t f ( t ) t λ {\displaystyle \lim _{t\to \infty }{\frac {f(t)}{t^{\lambda }}}} existe e lim s 0 s λ + 1 F ( s ) {\displaystyle \lim _{s\downarrow 0}{s^{\lambda +1}F(s)}} existe então

lim t f ( t ) t λ = 1 Γ ( λ + 1 ) lim s 0 s λ + 1 F ( s ) {\displaystyle \lim _{t\to \infty }{\frac {f(t)}{t^{\lambda }}}={\frac {1}{\Gamma (\lambda +1)}}\lim _{s\downarrow 0}{s^{\lambda +1}F(s)}}

Onde Γ ( x ) {\displaystyle \Gamma (x)} denota a função Gamma .[5]

Formulários

Teoremas de valor final para obtenção lim t f ( t ) {\displaystyle \lim _{t\to \infty }f(t)} têm aplicações no estabelecimento da estabilidade de longo prazo de um sistema .

Deduzindo lim s 0 s F ( s ) {\displaystyle \lim _{s\,\to \,0}{sF(s)}}

Supondo que f : ( 0 , ) C {\displaystyle f:(0,\infty )\to \mathbb {C} } é limitado e mensurável e lim t f ( t ) = α C {\displaystyle \lim _{t\to \infty }f(t)=\alpha \in \mathbb {C} } . Então F ( s ) {\displaystyle F(s)} existe para todo s > 0 {\displaystyle s>0} e lim s 0 + s F ( s ) = α {\displaystyle \lim _{s\,\to \,0^{+}}{sF(s)}=\alpha } .[7]

Supondo por conveniência que | f ( t ) | 1 {\displaystyle |f(t)|\leq 1} em ( 0 , ) {\displaystyle (0,\infty )} , e fazendo p α = lim t f ( t ) {\displaystyle \alpha =\lim _{t\to \infty }f(t)} . Se ϵ > 0 {\displaystyle \epsilon >0} e escolhendo A {\displaystyle A} de modo a | f ( t ) α | < ϵ {\displaystyle |f(t)-\alpha |<\epsilon } para todos t > A {\displaystyle t>A} . Como s 0 e s t d t = 1 {\displaystyle s\int _{0}^{\infty }e^{-st}\,dt=1} , para cada s > 0 {\displaystyle s>0} temos

s F ( s ) α = s 0 ( f ( t ) α ) e s t d t ; {\displaystyle sF(s)-\alpha =s\int _{0}^{\infty }(f(t)-\alpha )e^{-st}\,dt;}

logo

| s F ( s ) α | s 0 A | f ( t ) α | e s t d t + s A | f ( t ) α | e s t d t 2 s 0 A e s t d t + ϵ s A e s t d t = I + I I . {\displaystyle |sF(s)-\alpha |\leq s\int _{0}^{A}|f(t)-\alpha |e^{-st}\,dt+s\int _{A}^{\infty }|f(t)-\alpha |e^{-st}\,dt\leq 2s\int _{0}^{A}e^{-st}\,dt+\epsilon s\int _{A}^{\infty }e^{-st}\,dt=I+II.}

Agora para todo s > 0 {\displaystyle s>0} temos

I I < ϵ s 0 e s t d t = ϵ {\displaystyle II<\epsilon s\int _{0}^{\infty }e^{-st}\,dt=\epsilon } .

Por outro lado, desde A < {\displaystyle A<\infty } é fixo, é claro que lim s 0 I = 0 {\displaystyle \lim _{s\to 0}I=0} , e então | s F ( s ) α | < ϵ {\displaystyle |sF(s)-\alpha |<\epsilon } E se s > 0 {\displaystyle s>0} é pequeno o suficiente.

Supondo que todas as seguintes condições sejam satisfeitas:

  1. f : ( 0 , ) C {\displaystyle f:(0,\infty )\to \mathbb {C} } é continuamente diferenciável e ambos f {\displaystyle f} e f {\displaystyle f'} tem uma transformação de Laplace
  2. f {\displaystyle f'} é absolutamente integrável, isto é 0 | f ( τ ) | d τ {\displaystyle \int _{0}^{\infty }|f'(\tau )|\,d\tau } é finito
  3. lim t f ( t ) {\displaystyle \lim _{t\to \infty }f(t)} existe e é finito

Então

lim s 0 + s F ( s ) = lim t f ( t ) {\displaystyle \lim _{s\to 0^{+}}sF(s)=\lim _{t\to \infty }f(t)} .[8]

Observação

A prova usa o Teorema da Convergência Dominada .[8]

Teorema do valor final para somas assintóticas de funções periódicas

Deixei f : ( 0 , ) C {\displaystyle f:(0,\infty )\to \mathbb {C} } ser uma função contínua e limitada de modo que exista o seguinte limite

lim T 1 T 0 T f ( t ) d t = α C {\displaystyle \lim _{T\to \infty }{\frac {1}{T}}\int _{0}^{T}f(t)\,dt=\alpha \in \mathbb {C} }

Então lim s 0 , s > 0 s F ( s ) = α {\displaystyle \lim _{s\,\to \,0,\,s>0}{sF(s)}=\alpha } .[9]

Suponha que f : [ 0 , ) R {\displaystyle f:[0,\infty )\to \mathbb {R} } é contínuo e absolutamente integrável em [ 0 , ) {\displaystyle [0,\infty )} . Suponha ainda que f {\displaystyle f} é assintoticamente igual a uma soma finita de funções periódicas f a s {\displaystyle f_{\mathrm {as} }} , isso é

| f ( t ) f a s ( t ) | < ϕ ( t ) {\displaystyle |f(t)-f_{\mathrm {as} }(t)|<\phi (t)}

Onde ϕ ( t ) {\displaystyle \phi (t)} é absolutamente integrável em [ 0 , ) {\displaystyle [0,\infty )} e desaparece no infinito. Então

lim s 0 s F ( s ) = lim t 1 t 0 t f ( x ) d x {\displaystyle \lim _{s\to 0}sF(s)=\lim _{t\to \infty }{\frac {1}{t}}\int _{0}^{t}f(x)\,dx} .[10]

Definindo f ( t ) : [ 0 , ) R {\displaystyle f(t):[0,\infty )\to \mathbb {R} } e tal que F ( s ) {\displaystyle F(s)} seja a transformação de Laplace de f ( t ) {\displaystyle f(t)} . Supondo que f ( t ) {\displaystyle f(t)} satisfaz todas as seguintes condições:

  1. f ( t ) {\displaystyle f(t)} é infinitamente diferenciável em zero
  2. f ( k ) ( t ) {\displaystyle f^{(k)}(t)} tem uma transformação de Laplace para todos os inteiros não negativos k {\displaystyle k}
  3. f ( t ) {\displaystyle f(t)} diverge para o infinito como t {\displaystyle t\to \infty }

Então s F ( s ) {\displaystyle sF(s)} diverge para o infinito como s 0 + {\displaystyle s\to 0^{+}} .[11]

Teoremas de valor final para obtenção lim s 0 s F ( s ) {\displaystyle \lim _{s\,\to \,0}{sF(s)}} tem aplicações em probabilidade e estatística para calcular os momentos de uma variável aleatória . Sendo R ( x ) {\displaystyle R(x)} uma função de distribuição cumulativa de uma variável aleatória contínua X {\displaystyle X} e tomando ρ ( s ) {\displaystyle \rho (s)} como a transformada Laplace-Stieltjes de R ( x ) {\displaystyle R(x)} . Então o n {\displaystyle n} -ésimo momento de X {\displaystyle X} pode ser calculado como

E [ X n ] = ( 1 ) n d n ρ ( s ) d s n | s = 0 {\displaystyle E[X^{n}]=(-1)^{n}\left.{\frac {d^{n}\rho (s)}{ds^{n}}}\right|_{s=0}}

A estratégia é escrever

d n ρ ( s ) d s n = F ( G 1 ( s ) , G 2 ( s ) , , G k ( s ) , ) {\displaystyle {\frac {d^{n}\rho (s)}{ds^{n}}}={\mathcal {F}}{\bigl (}G_{1}(s),G_{2}(s),\dots ,G_{k}(s),\dots {\bigr )}}

Onde F ( ) {\displaystyle {\mathcal {F}}(\dots )} é contínuo e para cada k {\displaystyle k} , G k ( s ) = s F k ( s ) {\displaystyle G_{k}(s)=sF_{k}(s)} para uma função F k ( s ) {\displaystyle F_{k}(s)} . Para cada k {\displaystyle k} , definindo f k ( t ) {\displaystyle f_{k}(t)} como a transformada de Laplace inversa de F k ( s ) {\displaystyle F_{k}(s)} , obtemos lim t f k ( t ) {\displaystyle \lim _{t\to \infty }f_{k}(t)} , e aplicando um teorema do valor final podemos deduzir lim s 0 G k ( s ) = lim s 0 s F k ( s ) = lim t f k ( t ) {\displaystyle \lim _{s\,\to \,0}{G_{k}(s)}=\lim _{s\,\to \,0}{sF_{k}(s)}=\lim _{t\to \infty }f_{k}(t)} . Desta forma

d n ρ ( s ) d s n | s = 0 = F ( lim s 0 G 1 ( s ) , lim s 0 G 2 ( s ) , , lim s 0 G k ( s ) , ) {\displaystyle \left.{\frac {d^{n}\rho (s)}{ds^{n}}}\right|_{s=0}={\mathcal {F}}{\Bigl (}\lim _{s\,\to \,0}G_{1}(s),\lim _{s\,\to \,0}G_{2}(s),\dots ,\lim _{s\,\to \,0}G_{k}(s),\dots {\Bigr )}}

Exemplos

Exemplo onde FVT se mantém

Por exemplo, para um sistema descrito pela função de transferência

H ( s ) = 6 s + 2 , {\displaystyle H(s)={\frac {6}{s+2}},}

e assim a resposta ao impulso converge para

lim t h ( t ) = lim s 0 6 s s + 2 = 0. {\displaystyle \lim _{t\to \infty }h(t)=\lim _{s\,\searrow \,0}{\frac {6s}{s+2}}=0.}

Ou seja, o sistema retorna a zero após ser perturbado por um curto impulso. No entanto, a transformada de Laplace da resposta ao degrau unitário é

G ( s ) = 1 s 6 s + 2 {\displaystyle G(s)={\frac {1}{s}}{\frac {6}{s+2}}}

e assim a resposta ao degrau converge para

lim t g ( t ) = lim s 0 s s 6 s + 2 = 6 2 = 3 {\displaystyle \lim _{t\to \infty }g(t)=\lim _{s\,\searrow \,0}{\frac {s}{s}}{\frac {6}{s+2}}={\frac {6}{2}}=3}

e assim um sistema de estado zero seguirá um aumento exponencial para um valor final de 3.

Exemplo onde FVT não se mantém

Para um sistema descrito pela função de transferência

H ( s ) = 9 s 2 + 9 , {\displaystyle H(s)={\frac {9}{s^{2}+9}},}

o teorema do valor final parece prever o valor final da resposta ao impulso como sendo 0 e o valor final da resposta ao degrau sendo 1. No entanto, nenhum limite no domínio do tempo existe e, portanto, as previsões do teorema do valor final não são válidas. Na verdade, tanto a resposta ao impulso quanto a resposta ao degrau oscilam e (neste caso especial) o teorema do valor final descreve os valores médios em torno dos quais as respostas oscilam.

Existem duas verificações realizadas na teoria de controle que confirmam resultados válidos para o Teorema do Valor Final:

  1. Todas as raízes diferentes de zero do denominador de H ( s ) {\displaystyle H(s)} deve ter partes reais negativas.
  2. H ( s ) {\displaystyle H(s)} não deve ter mais de um pólo na origem.

A regra 1 não foi satisfeita neste exemplo, em que as raízes do denominador são 0 + j 3 {\displaystyle 0+j3} e 0 j 3 {\displaystyle 0-j3} .

Teoremas de valor final para a transformada Z

Deduzindo lim k f [ k ] {\displaystyle \lim _{k\to \infty }f[k]}

E se lim k f [ k ] {\displaystyle \lim _{k\to \infty }f[k]} existe e lim z 1 ( z 1 ) F ( z ) {\displaystyle \lim _{z\,\to \,1}{(z-1)F(z)}} existe então lim k f [ k ] = lim z 1 ( z 1 ) F ( z ) {\displaystyle \lim _{k\to \infty }f[k]=\lim _{z\,\to \,1}{(z-1)F(z)}} . [4]:101

Ver também

  • Teorema do valor inicial
  • Transformada Z
  • Laplace Transform

Notas

  1. Wang, Ruye (17 de fevereiro de 2010). «Initial and Final Value Theorems». Consultado em 21 de outubro de 2011 
  2. Alan V. Oppenheim; Alan S. Willsky; S. Hamid Nawab (1997). Signals & Systems. Prentice Hall. New Jersey, USA: [s.n.] ISBN 0-13-814757-4 
  3. a b c Schiff, Joel L. (1999). The Laplace Transform: Theory and Applications. Springer. New York: [s.n.] ISBN 978-1-4757-7262-3 
  4. a b c d Graf, Urs (2004). Applied Laplace Transforms and z-Transforms for Scientists and Engineers. Birkhäuser Verlag. Basel: [s.n.] ISBN 3-7643-2427-9 
  5. a b c Chen, Jie; Lundberg, Kent H.; Davison, Daniel E.; Bernstein, Dennis S. (Junho de 2007). «The Final Value Theorem Revisited - Infinite Limits and Irrational Function». IEEE Control Systems Magazine. 27: 97-99. doi:10.1109/MCS.2007.365008 
  6. «Final Value Theorem of Laplace Transform». ProofWiki. Consultado em 12 de abril de 2020 
  7. a b Ullrich, David C. (26 de maio de 2018). «The tauberian final value Theorem». Math Stack Exchange 
  8. a b Sopasakis, Pantelis (18 de maio de 2019). «A proof for the Final Value theorem using Dominated convergence theorem». Math Stack Exchange 
  9. Murthy, Kavi Rama (7 de maio de 2019). «Alternative version of the Final Value theorem for Laplace Transform». Math Stack Exchange 
  10. Gluskin, Emanuel (1 de novembro de 2003). «Let us teach this generalization of the final-value theorem». European Journal of Physics. 24: 591–597. doi:10.1088/0143-0807/24/6/005 
  11. Hew, Patrick (22 de abril de 2020). «Final Value Theorem for function that diverges to infinity?». Math Stack Exchange 

Ligações externas

  • [1]
  • http://fourier.eng.hmc.edu/e102/lectures/Laplace_Transform/node17.html Valor final para Laplace
  • https://web.archive.org/web/20110719222313/http://www.engr.iupui.edu/~skoskie/ECE595s7/handouts/fvt_proof.pdf Prova de valor final para Z-transformações